Question

Consider a firm that has cost function of: TC = 24 + 10Q + 1.5 Q^2...

Consider a firm that has cost function of:

TC = 24 + 10Q + 1.5 Q^2

a) Draw the average cost and marginal cost curves for this firm (label with equation).

b) Draw the supply curve of this firm

c) What are the profits of the firm if the price of output is $44?

0 0
Add a comment Improve this question Transcribed image text
Answer #1

Answer

The red line is the MC curve

The blue line is the AC curve

MC=dTC/dQ
MC= d(24 + 10Q + 1.5 Q^2)dQ
MC=10+3Q

AC=TC/Q
AC=(24 + 10Q + 1.5 Q^2)/Q
AC=24/Q+10+1.5Q

10+ 3x AC & MC 24/x + 10 +1 undefined 35.5 25 0 quantity

b)


The rising portion of the MC above the minimum of the AVC is the short-run supply curve of the firm
TC=24 + 10Q + 1.5 Q^2
FC=24
VC=10Q + 1.5 Q^2
AVC=VC/Q
AVC=(10Q + 1.5 Q^2)/Q
AVC=10+1.5Q

The red line is the supply curve of the firm

AVC & MC 10 + 3x 10 + 1.5x 10 11.5 quantityc)

The firm produces at MC=P
MC=10+3Q
P=44
10+3Q=44
3Q=34
Q=11.33
Profit =TR-TC
TR=P*Q=44*11.33=498.52
TC=24+10*11.33+1.5*(11.33^2)
=329.85335
Profit=498.52-329.85335
=168.66665
=169
the profit is $169

Add a comment
Know the answer?
Add Answer to:
Consider a firm that has cost function of: TC = 24 + 10Q + 1.5 Q^2...
Your Answer:

Post as a guest

Your Name:

What's your source?

Earn Coins

Coins can be redeemed for fabulous gifts.

Not the answer you're looking for? Ask your own homework help question. Our experts will answer your question WITHIN MINUTES for Free.
Similar Homework Help Questions
  • Consider a competitive rm with total costs given by TC(q) = 100 + 10q + q^2,...

    Consider a competitive rm with total costs given by TC(q) = 100 + 10q + q^2, The firm faces a market price p = 50. (a) Write expressions for total revenue TR and marginal revenue MR as functions of output q. (b) Write expressions for average total cost ATC, average variable cost AVC, and marginal cost MC as functions of output q. (c) For what value of output is ATC minimized? (d) Find the profit maximizing level of output q...

  • Consider a competitive firm with total costs given by TC(q) = 100 + 10q + q...

    Consider a competitive firm with total costs given by TC(q) = 100 + 10q + q 2 The firm faces a market price p = 50. (d) Find the profit-maximizing level of output q^*. At this level of output, what are TR, TC, ATC, and π? (e) Graph the ATC, AVC, MC, and MR curves in a single graph, and indicate the profit-maximizing level of output. If there are profits, shade the region corresponding to profit and label it.

  • Consider a competitive firm with total costs given by TC(q) = 100 + 10q + q...

    Consider a competitive firm with total costs given by TC(q) = 100 + 10q + q 2 The firm faces a market price p = 50. (a) Write expressions for total revenue TR and marginal revenue MR as functions of output q. (b) Write expressions for average total cost ATC, average variable cost AVC, and marginal cost MC as functions of output q. (c) For what value of output is ATC minimized?

  • The market price is p=50 3. Consider a competitive firm with total costs given by TC(q)...

    The market price is p=50 3. Consider a competitive firm with total costs given by TC(q) = 100 + 10q+q? (e) Graph the ATC, AVC, MC, and MR curves in a single graph, and indicate the profit maximizing level of output. If there are profits, shade the region corre- sponding to profit and label it. (f) If fixed costs increase from 100 to 500, what happens to the profit maximizing level of output, TR, TC, and a? (g) If fixed...

  • Suppose that the cost function of a firm is C(Q) = 490 + 10Q^2 . (a)...

    Suppose that the cost function of a firm is C(Q) = 490 + 10Q^2 . (a) Provide the mathematical expressions for AFC, AVC, AC and MC. (b) Graph all the costs above as a function of Q. (c) What is true about the relation between the marginal cost and the average costs when the latter are at their minimum? And when the average cost are increasing (decreasing)? Explain.

  • Suppose a firm has the following total cost function: TC = 300 + 40 Q –...

    Suppose a firm has the following total cost function: TC = 300 + 40 Q – 8Q2 + (2/3) Q3 (a) Write an equation for (i) average fixed cost; and (ii) average variable cost (b) What will be the value of average (total) cost when Q = 60? (c) What will be the marginal cost, when Q = 20? (d) For this firm what will be the value of average variable cost at its minimum

  • Question 4 Consider the Sunshine Company, a perfectly competitive firm with the following cost function TC...

    Question 4 Consider the Sunshine Company, a perfectly competitive firm with the following cost function TC 12006Q + 202 where Q is the firm's output per day. a) Find the firm's marginal cost function. [2 marks] C b) If the price of Sunshine's product equals $66, how many units per day should the firm produce? [4 marks] c) Find the firm's average variable cost function. [3 marks] d) Is average variable cost at the quantity you calculated in part b)...

  • Consider a competitive firm with total costs given by TC(q) = 100 + 10q + q 2 The firm faces a market price p = 50. (f)...

    Consider a competitive firm with total costs given by TC(q) = 100 + 10q + q 2 The firm faces a market price p = 50. (f) If fixed costs increase from 100 to 500, what happens to the profit-maximizing level of output, TR, TC, and π? (g) If fixed costs increase from 100 to 500, should the firm continue to operate in the short-run? What about the long-run?

  • 2. An industry consists of many identical firms, each with the cost function C(q) = 100...

    2. An industry consists of many identical firms, each with the cost function C(q) = 100 + 30q – 8q2 + q3 a. Derive the average cost, average variable cost, and marginal cost curves of a firm. b. Compute the outputs at which the AC and AVC curves reach their minimums. C. If the market price is $40, and each firm is a price-taker, how much output will each firm supply? d. How much profit or loss is each firm...

  • Suppose that a firm has a short run, total cost function given by: TC= 1089 +10q...

    Suppose that a firm has a short run, total cost function given by: TC= 1089 +10q +9q2. 1. Determine the profit-maximizing quantity of production when price is $244. _____________________________________ q= 13 2. Calculate the price at which this firm breaks even (i.e. profit = $0). _____________________________________ $208 3. Calculate the price at which this firm shuts down in the short run. _____________________________________ $10 The answers are given but can you show how to get them step by step.

ADVERTISEMENT
Free Homework Help App
Download From Google Play
Scan Your Homework
to Get Instant Free Answers
Need Online Homework Help?
Ask a Question
Get Answers For Free
Most questions answered within 3 hours.
ADVERTISEMENT
ADVERTISEMENT
ADVERTISEMENT